The density of atmosphere (measured in kilograms/meter3) on a certain planet is found to decrease as altitude increases (as measured from the planet's surface). What type of relationship exists between the altitude and the atmospheric density, and what would the atmospheric density be at an altitude of 1,291 kilometers?


A.
inverse plot, 0.45 kilograms/meter3
B.
inverse plot, 0.51 kilograms/meter3
C.
quadratic plot, 1.05 kilograms/meter3
D.
inverse plot, 1.23 kilograms/meter3
E.
inverse plot, 0.95 kilograms/meter3

The Density Of Atmosphere (measured In Kilograms/meter3) On A Certain Planet Is Found To Decrease As

Answers

Answer 1

' A ' looks like the best choice.

Answer 2

Answer:

B.  inverse plot, 0.51 kilograms/meter3

Explanation:


Related Questions

determine the smallest mass of lead that when tied using a string to a wooden boat on a pond will be enough to sink the toy boat. assuming specific gravity of wood is 0.5 and density of water is 1000kg per cubic metre?​

Answers

The mass is going to be about 100m/s

For the RC circuit and the RL circuit, assume that the period of the source square wave is much larger than the time constant for each. Make a sketch of vR(t) as a function of t for each of the circuits?

Answers

Answer with Explanation:

Concepts and reason

The concept to solve this problem is that if a capacitor is connected in a RC circuit then it allows the flow of charge through circuit only till it gets fully charged. Once the capacitor is charged it will not allow any charge or current to flow.

Opposite is the case with inductor in the RL circuit. According to Faraday's law an inductor develops an emf to oppose the voltage applied but once the flux change stops then the inductor behaves just like a normal wire as if no inductor is there.

In attached figure, resistor is connected in series to the capacitor.

As we considered [tex]V_{C}[/tex] the voltage across the capacitor and [tex]V_{s}[/tex] the voltage across the source.

Voltage across a resistor In RC circuit.

[tex]V_{R}=V_S\left ( e^{-\frac{t}{RC}} \right )[/tex]

Voltage across a resistor In RL circuit.

[tex]V_{R}=V_S\left (1- e^{-\frac{Rt}{L}} \right )[/tex]

The sketch of [tex]\mathbf{v_R(t)}[/tex] as a function of t for each of the circuits can be seen in the diagram attached below.

For the Pre-Laboratory exercise, based on the assumption that the RC circuit has a capacitor and a sensing resistor while the RL circuit has a sensing resistor and an inductor.

The input voltage for both circuits is regarded as the square wave and if the square wave is much larger than the time constant for each.

Therefore, we can conclude that the below diagram shows an appropriate sketch of  [tex]\mathbf{v_R(t)}[/tex] as a function of t for each of the circuits.

Learn more about RC circuits and RL circuits here:

https://brainly.com/question/15595203

A worker pushes on a crate that experiences a net force of 45.0 N. If it accelerates at 0.500 m/s2 what is the weight?

Answers

Answer:

882 N

Explanation:

F = ma

45.0 N = m (0.500 m/s²)

m = 90.0 kg

mg = 882 N

Two workers are sliding 330 kg crate across the floor. One worker pushes forward on the crate with a force of 430 N while the other pulls in the same direction with a force of 330 N using a rope connected to the crate. Both forces are horizontal, and the crate slides with a constant speed. What is the crate's coefficient of kinetic friction on the floor?

Answers

Answer:

Coefficient of kinetic friction = 0.235

Explanation:

Given:

Mass of crate = 330 kg

1st force = 430 N

2nd force = 330 N

Find:

Coefficient of kinetic friction.

Computation:

We know that, velocity is constant.

So, acceleration (a) = 0

So, net force (f) = 430 N + 330 N

Net force (f) = 760 N

F = μmg

μ = f / mg                                   [∵ g = 9.8]

μ = 760 / [330 × 9.8]

μ = 760 / [3,234]

μ = 0.235

Coefficient of kinetic friction = 0.235

The NASA spacecraft Deep Space I was shut down on December 18, 2001, following a three-year journey to the asteroid Braille and the comet Borrelly. This spacecraft used a solar-powered ion engine to produce 0.064 ounces of thrust (force) by stripping electrons from neon atoms and accelerating the resulting ions to 70,000 mi/h. The thrust was only as much as the weight of a couple sheets of paper, but the engine operated continuously for 16,000 hours. As a result, the speed of the spacecraft increased by 7900 mi/h. What was the mass of Deep Space I?

Answers

Answer:

The mass will be "8.86 lb".

Explanation:

The given values are:

Force

= 70,000 mi/h

Speed

= 7900 mi/h

On applying the Law of momentum, we get

⇒  [tex]V_{1}m_{1}=V_{2}m_{2}[/tex]

On putting the estimated values, we get

⇒  [tex]70000 = 7900\times mass \ of \ deepspace \ 1[/tex]

⇒  [tex]mass \ of \ deepspace \ 1 = \frac{70000}{7900}[/tex]

⇒                                    [tex]=8.86 \ lb[/tex]

What is the velocity of a car that travels 556km northwest in 3.2 hours

Answers

Answer:

173.75 km/hr in the NW direction.

Explanation:

Velocity is the time rate of change in displacement of a body. Mathematically:

v = d / t

where d = displacement

t = time

Therefore, the velocity of the car is:

v = 556 / 3.2 = 173.75 km/hr

The velocity of the car is 173.75 km/hr in the NW direction.

The velocity of a car will be "173.75 km/hr".

Displacement and Velocity,

The velocity of something like a car moving northward on something like a prominent motorway as well as the velocity of something like a rocket launching towards spacecraft both might be determined or monitored.

Displacement, d = 556 km

Time, t = 3.2 hours

We know the relation,

→ Velocity = [tex]\frac{Displacement}{Time}[/tex]

or,

→ V = [tex]\frac{d}{t}[/tex]

By substituting the values, we get

      = [tex]\frac{556}{3.2}[/tex]

      = [tex]173.75[/tex] km/hr

Thus the response above is right.

Find out more information about velocity here:

https://brainly.com/question/6504879

How can I show that the sphere of radius R performs a simple harmonic movement. how can i set its reference point and make the free body diagram.

I have the torque sum equation which is equal to the moment of inertia by angular acceleration

Answers

Explanation:

Draw a free body diagram of the pendulum (the combination of the sphere and the massless rod).  There are three forces on the pendulum:

Weight force mg at the center of the sphere,

Reaction force in the x direction at the pivot,

Reaction force in the y direction at the pivot.

Sum the torques about the pivot O.

∑τ = I d²θ/dt²

mg (L sin θ) = I d²θ/dt²

For small θ, sin θ ≈ θ.

mg L θ = I d²θ/dt²

Since d²θ/dt² is directly proportional to θ, this fits the definition of simple harmonic motion.

If you wish, you can use parallel axis theorem to find the moment of inertia about O:

I = Icm + md²

I = ⅖ mr² + mL²

mg L θ = (⅖ mr² + mL²) d²θ/dt²

gL θ = (⅖ r² + L²) d²θ/dt²

A note on a piano vibrates 262 times per second . What is the period of the wave ?

Answers

it gets 5 beats per minute.

510 g squirrel with a surface area of 935 cm2 falls from a 4.8-m tree to the ground. Estimate its terminal velocity. (Use the drag coefficient for a horizontal skydiver. Assume that the squirrel can be approximated as a rectanglar prism with cross-sectional area of width 11.6 cm and length 23.2 cm. Note, the squirrel may not reach terminal velocity by the time it hits the gr

Answers

Answer:

The terminal velocity is [tex]v_t =17.5 \ m/s[/tex]

Explanation:

From the question we are told that

       The mass of the squirrel is  [tex]m_s = 50\ g = \frac{50}{1000} = 0.05 \ kg[/tex]

      The surface area is   [tex]A_s = 935 cm^2 = \frac{935}{10000} = 0.0935 \ m^2[/tex]

       The height of fall is  h =4.8 m

        The length of the prism is [tex]l = 23.2 = 0.232 \ m[/tex]

          The width of the prism is [tex]w = 11.6 = 0.116 \ m[/tex]

 

The terminal velocity is mathematically represented as

       [tex]v_t = \sqrt{\frac{2 * m_s * g }{\dho_s * C * A } }[/tex]

Where [tex]\rho[/tex]  is the density of a rectangular prism with a constant values of [tex]\rho = 1.21 \ kg/m^3[/tex]

            [tex]C[/tex] is the drag coefficient for a horizontal skydiver with a value = 1

            A  is the area of the prism the squirrel is assumed to be which is mathematically represented as

      [tex]A = 0.116 * 0.232[/tex]

       [tex]A = 0.026912 \ m^2[/tex]

 substituting values

      [tex]v_t = \sqrt{\frac{2 * 0.510 * 9.8 }{1.21 * 1 * 0.026912 } }[/tex]

     [tex]v_t =17.5 \ m/s[/tex]

       

You drive in straight line at 20 m/s for 10 miles, then at 30m/s for an other 10 miles what is your average speed

Answers

Answer:

25 m/s

Explanation:

Data provided in the question

20 m/s for 10 minutes

And, the 30 m/s for another 10 minutes

Based on the above information, the average speed is

As we know that

[tex]Average\ speed = \frac{Total\ distance}{Total\ time}[/tex]

[tex]= \frac{20\times10\times60 + 30\times10\times60 }{20\times60}[/tex]

= 25 m/s

1 hour = 60 minutes

1 minute = 60 seconds

Hence, the average speed is 25 m/s

In the question,  there are miles is given but instead of this we use the minutes as we have to find out the average speed and time should not be in miles it should be in minutes, hour or seconds

Therefore we considered the same

A projectile is defined as

Answers

Answer:

By definition, a projectile has a single force that acts upon it - the force of gravity.

Explanation:

A projectile is any object that once projected or dropped continues in motion by its own inertia and is influenced only by the downward force of gravity.

// have a great day //

Which person will most likely hear the loudest sound?

A
B
C
D

Answers

Answer:

The youngest person

Explanation:

Hearing worsens with age

Please mark brainliest

Answer:

A

Explanation:

The person closest to the origin of the sound will most likely hear the loudest sound. ^^

Although these quantities vary from one type of cell to another, a cell can be 2.2 micrometers in diameter with a cell wall 40 nm thick. If the density (mass divided by volume) of the wall material is the same as that of pure water, what is the mass (in mg) of the cell wall, assuming the cell to be spherical and the wall to be a very thin spherical shell?

Answers

Answer:

m = 6.082 x 10⁻¹⁶ kg = 6.082 x 10⁻¹⁰ mg

Explanation:

First, we find the the surface area of the cell wall. Since, the cell is spherical in shape. Therefore, surface area of cell wall will be:

A = 4πr²

where,

A = Surface Area = ?

r = Radius of Cell = Diameter/2 = 2.2 μm/2 = 1.1 μm = 1.1 x 10⁻⁶ m

Therefore,

A = 4π(1.1 x 10⁻⁶ m)²

A = 15.2 x 10⁻¹² m²

Now, we find the volume of the cell wall. For that purpose, we use formula:

V = At

where,

V = Volume of the Cell Wall = ?

t = Thickness of Wall = 40 nm = 4 x 10⁻⁸ m

Therefore,

V = (15.2 x 10⁻¹² m²)(4 x 10⁻⁸ m)

V = 60.82 x 10⁻²⁰ m³

Now, to find mass of cell wall, we use formula:

ρ = m/V

m = ρV

where,

ρ = density of water = 1000 kg/m³

m = Mass of Wall = ?

Therefore,

m = (1000 kg/m³)(60.82 x 10⁻²⁰ m³)

m = 6.082 x 10⁻¹⁶ kg = 6.082 x 10⁻¹⁰ mg

The mass of the cell wall in mg is 6.082 × 10⁻¹⁰ mg

Since we assume the cell to be spherical and the wall to be a thin spherical shell, the volume of the cell wall V = At where

A = surface area of cell = 4πR² where R = radius of cell = 2.2 μm/2 = 1.1 × 10⁻⁶ m and t = thickness of cell wall = 40 nm = 40 × 10⁻⁹ m.Volume of cell wall

So, V = 4πR²t

Substituting the values of the variables into the equation, we have

V = 4πR²t

V = 4π(1.1 × 10⁻⁶ m)² × 40 × 10⁻⁹ m.

V = 4π(1.21 × 10⁻¹² m²) × 40 × 10⁻⁹ m.

V = 193.6π × 10⁻²¹ m³

V = 608.21 × 10⁻²¹ m³

V = 6.0821 × 10⁻¹⁹ m³

V ≅ 6.082 × 10⁻¹⁹ m³

Mass of the cell wall

We know that density of cell wall, ρ = m/v where m = mass of cell wall and V = volume of cell wall.

Making m subject of the formula, we have

m = ρV

Since we assume the density of the cell wall to be equal to that of pure water, ρ = 1000 kg/m³

So, m = ρV

m = 1000 kg/m³ × 6.082 × 10⁻¹⁹ m³

m = 6.082 × 10⁻¹⁶ kg

Converting to mg, we have

m = 6.082 × 10⁻¹⁶ kg × 10⁶ mg/kg

m = 6.082 × 10⁻¹⁰ mg

So, the mass of the cell wall in mg is 6.082 × 10⁻¹⁰ mg

Learn more about mass of cell wall here:

https://brainly.com/question/13173768

If you secure a refrigerator magnet about 2mmfrom the metallic surface of a refrigerator door and then move the magnet sideways, you can feel a resistive force, indicating the presence of eddy currents in the surface.

A)Estimate the magnetic field strength Bof the magnet to be 5 mTand assume the magnet is rectangular with dimensions 4 cmwide by 2 cmhigh, so its area A is 8 cm2. Now estimate the magnetic flux ΦB into the refrigerator door behind the magnet.
Express your answer with the appropriate units.

B)If you move the magnet sideways at a speed of 2 cm/s, what is a corresponding estimate of the time rate at which the magnetic flux through an area A fixed on the refrigerator is changing as the magnet passes over? Use this estimate to estimate the emf induced under the rectangle on the door's surface.
Express your answer with the appropriate units.

Answers

Answer:

(A) 4* 6 ^ ⁻6 T m² (B) 2 * 10 ^ ⁻6 v

Explanation:

Solution

Given that:

A refrigerator magnet about = 2 mm

The estimated magnetic field strength of the magnet is = 5 m T

The Area = 8 cm²

Now,

(A) The magnetic flux ΦB = BA

Thus,

ΦB  = (5 * 10^⁻ 3) ( 4 * 10 ^⁻2) * ( 2 * 10^ ⁻2) Tm²

So,

ΦB =  4* 6 ^ ⁻6 T m²

(B)By applying Faraday's Law we have the following formula given below:

Ε = Bℓυ

Here,

ℓ = 2 cm the same as 2 * 10 ^⁻2 m

B = 5 m T = 5 * 10 ^ ⁻3 T

υ = 2 cm/s  = 2 * 10 ^ ⁻2 m/s

Thus,

Ε = (5 * 10 ^ ⁻3 T) *  (2 * 10 ^ ⁻2) (2 * 10 ^ ⁻2) v

E =2 * 10 ^ ⁻6 v

A) The magnetic flux ΦB into the refrigerator door behind the magnet :

4 * 6⁻⁶ Tm²

B) The estimated emf induced under the rectangle on the door's surface ;

2 * 10⁻⁶ v

Given data :

magnetic field strength of magnet ( B )  = 5 mT

size of refrigerator magnet = 2 mm

Area of magnet ( A )  = 4 * 2 = 8 cm²

A) Determine the magnetic flux ΦB

where ; ΦB  = BA

ΦB = ( 5 * 10⁻³ ) * ( 4 * 10⁻² ) * ( 2 * 10⁻² ) Tm²

      =  4 * 6⁻⁶ Tm²

B) Determine estimated emf induced

To determine the estimated emf we will apply Faraday's law

Ε = Bℓυ ---- ( 2 )

where : B = 5 * 10⁻³ T,  ℓ = 2 * 10⁻² m,  υ = 2 * 10⁻² m/s

insert values into equation 2

E = ( 5 * 10⁻³ ) * ( 2 * 10⁻² ) * ( 2 * 10⁻² )

  = 2 * 10⁻⁶ v

Hence we can conclude that The magnetic flux ΦB is 4 * 6⁻⁶ Tm² and The estimated emf induced is  2 * 10⁻⁶ v

Learn more about magnet flux : https://brainly.com/question/4721624

Why do bears activity increase as certain points during the day

Answers

Because they are well rested and have to work to get food in their system.

When solving vector addition problems you can use either the graphical
method or the

Answers

Answer :the resultant of two vectors can be found using either the parallelogram method or the triangle method. don't know if this was helpful ?

Explanation:

Answer:

Analytical method.

An experiment invilves three charges objects: A, B, and C. Object A repels object B and attracts onject C. object C ir repelled by ebonite charged with fur. What is the charge on the object?

Answers

Answer:

A and B is positive charge

C_negative

Explanation:

because when an ebonite is rubbed with fur produce negative charge due to law of electrostatic like charge repel and unlike attract

An engine draws energy from a hot reservoir with a temperature of 1250 K and exhausts energy into a cold reservoir with a temperature of 322 K. Over the course of one hour, the engine absorbs 1.37 x 105 J from the hot reservoir and exhausts 7.4 x 104 J into the cold reservoir.

1) What is the power output of this engine?

2) What is the maximum (Carnot) efficiency of a heat engine running between these two reservoirs?

3) What is the actual efficiency of this engine?

Answers

Answer:

The power output of this engine is  [tex]P = 17.5 W[/tex]

The  the maximum (Carnot) efficiency is  [tex]\eta_c = 0.7424[/tex]

The  actual efficiency of this engine is  [tex]\eta _a = 0.46[/tex]

Explanation:

From the question we are told that

    The temperature of the hot reservoir is  [tex]T_h = 1250 \ K[/tex]

      The temperature of the cold reservoir  is  [tex]T_c = 322 \ K[/tex]

     The energy absorbed from the hot reservoir is [tex]E_h = 1.37 *10^{5} \ J[/tex]

       The energy exhausts into  cold reservoir is  [tex]E_c = 7.4 *10^{4} J[/tex]

The power output is mathematically represented as

      [tex]P = \frac{W}{t}[/tex]

Where t is the time taken which we will assume to be 1 hour =  3600 s  

W is the workdone which is mathematically represented as

      [tex]W = E_h -E_c[/tex]

substituting values

       [tex]W = 63000 J[/tex]

So

    [tex]P = \frac{63000}{3600}[/tex]

    [tex]P = 17.5 W[/tex]

The Carnot efficiency is mathematically represented as

          [tex]\eta_c = 1 - \frac{T_c}{T_h}[/tex]

         [tex]\eta_c = 1 - \frac{322}{1250}[/tex]

         [tex]\eta_c = 0.7424[/tex]

The actual efficiency is mathematically represented as

        [tex]\eta _a = \frac{W}{E_h}[/tex]

substituting values

         [tex]\eta _a = \frac{63000}{1.37*10^{5}}[/tex]

         [tex]\eta _a = 0.46[/tex]

     

g 95 N force exerted at the end of a 0.50 m long torque wrench gives rise to a torque of 15 N • m. What is the angle (assumed to be less than 90°) between the wrench handle and the direction of the applied force?

Answers

Answer:

Angle = 18.41°

Explanation:

Torque = F•r•sin θ

where;

F = force

r = distance from the rotation point

θ = the angle between the force and the radius vector.

We are given;

Torque = 15 N.m

F = 95 N

r = 0.5 m

Thus, plugging in the relevant values ;

15 = 95 × 0.5 × sin θ

sin θ = 15/(95 × 0.5)

sin θ = 0.3158

θ = sin^(-1)0.3158

θ = 18.41°

The amount of friction divided by the weight of an object forms a unit less number called the

Answers

Answer:

Coefficient of friction.

Explanation:

The amount of friction divided by the weight of an object is equal to the coefficient of friction. It is a dimensional less number. It can be given by :

[tex]F=\mu N[/tex]

N is normal force.

[tex]\mu[/tex] = coefficient of friction

[tex]\mu=\dfrac{F}{N}[/tex]

Q) Considering the value of ideal gas constant in S.I. unit, find the volume of 35g O2 at 27°C and 72
cm Hg pressure. Later, if we keep this pressure constant, the r.m.s velocity of this oxygen molecules
become double at a certain temperature. Calculate the value of this temperature.

Answers

Answer:

V = 0.0283 m³ = 28300 cm³

T₂ = 1200 K

Explanation:

The volume of the gas can be determined by using General Gas Equation:

PV = nRT

where,

P = Pressure of Gas = (72 cm of Hg)(1333.2239 Pa/cm of Hg) = 95992.12 Pa

V = Volume of Gas = ?

n = no. of moles = mass/molar mass = (35 g)/(32 g/mol) = 1.09 mol

R = General Gas Constant = 8.314 J/ mol.k

T = Temperature of Gas = 27°C + 273 = 300 k

Therefore,

(95992.12 Pa)(V) = (1.09 mol)(8.314 J/mol.k)(300 k)

V = 2718.678 J/95992.12 Pa

V = 0.0283 m³ = 28300 cm³

The Kinetic Energy of gas molecule is given as:

K.E = (3/2)(KT)

Also,

K.E = (1/2)(mv²)

Comparing both equations, we get:

(3/2)(KT) = (1/2)(mv²)

v² = 3KT/m

v = √(3KT/m)

where,

v = r.m.s velocity

K = Boltzamn Constant

T = Absolute Temperature

m = mass of gas molecule

At T₁ = 300 K, v = v₁

v₁ = √(3K*300/m)

v₁ = √(900 K/m)

Now, for v₂ = 2v₁ (double r.m.s velocity), T₂ = ?

v₂ = 2v₁ = √(3KT₂/m)

using value of v₁:

2√(900 K/m) = √(3KT₂/m)

4(900) = 3 T₂

T₂ = 1200 K

The “turning effect of a force” (T = F * r) is:
(a) determined as the product of force and the moment of inertia.
(b) generated by concentric forces.
(c) equivalent to the angular momentum.
(d) determined as a product of torque and moment arm.
(e) called “moment” or “torque”.

Answers

Answer:

b and e

Explanation:

r x F is the formula for torque.

The "turning effect" or torque happens when concentric forces rotate an object along said center.

a) False because T = Fr = Ia (a = angular acceleration)

b) True

c) False. L = Iw (w = angular velocity), which does not equal Ia

d) False. It is torque, not the product of torque and something else

e) True.

A 328-kg car moving at 19.1 m/s in the x direction hits from behind a second car moving at 13.0 m/s in the same direction. If the second car has a mass of 790 kg and a speed of 15.1 m/s right after the collision, what is the velocity of the first car after this sudden collision

Answers

Answer:

14.04 m/s

Explanation:

To find the velocity of the first car after the collision, we can use the equation of conservation of momentum:

m1v1 + m2v2 = m1'v1' + m2'v2'

We have the following data:

m1 = m1' = 328,

m2 = m2' = 790,

v1 = 19.1,

v2 = 13,

v2' = 15.1.

Using this data, we can find v1' (final velocity of the first car):

328 * 19.1 + 790 * 13 = 328 * v1' + 790 * 15.1

16534.8 = 328 * v1' + 11929

328 * v1' = 4605.8

v1' = 14.04 m/s

Olaf is standing on a sheet of ice that covers the football stadium parking lot in Buffalo, New York; there is negligible friction between his feet and the ice. A friend throws Olaf a ball of mass 0.400 kg that is traveling horizontally at 11.3 m/s. Olaf's mass is 75.0 kg. (a) If Olaf catches the ball, with what speed v_f do Olaf and the ball move afterward

Answers

Answer:

v = 0.059 m/s

Explanation:

To find the final speed of Olaf and the ball you use the conservation momentum law. The momentum of Olaf and the ball before catches the ball is the same of the momentum of Olaf and the ball after. Then, you have:

[tex]mv_{1i}+Mv_{2i}=(m+M)v[/tex]  (1)

m: mass of the ball = 0.400kg

M: mass of Olaf = 75.0 kg

v1i: initial velocity of the ball = 11.3m/s

v2i: initial velocity of Olaf = 0m/s

v: final velocity of Olaf and the ball

You solve the equation (1) for v and replace the values of all variables:

[tex]v=\frac{mv_{1i}}{m+M}=\frac{(0.400kg)(11.3m/s)}{0.400kg+75.0kg}=0.059\frac{m}{s}[/tex]

Hence, after Olaf catches the ball, the velocity of Olaf and the ball is 0.059m/s

Use Hooke's Law to determine the work done by the variable force in the spring problem. Nine joules of work is required to stretch a spring 0.5 meter from its natural length. Find the work required to stretch the spring an additional 0.40 meter.

Answers

Answer:

29.16 J

Explanation:

From Hook's law,

W = 1/2(ke²)..................... Equation 1

Where W = work done, k = Spring constant, e = extension.

Given: W = 9 J, e = 0.5 m.

Substitute into equation 1

9 = 1/2(k×0.5²)

Solve for k

k = 18/0.5²

k = 72 N/m.

The work done required to stretch the spring by additional 0.4 m is

W = 1/2(72)(0.4+0.5)²

W = 36(0.9²)

W = 29.16 J.

Volume of an block is 5 cm3. If the density of the block is 250 g/cm3, what is the mass of the block ?​

Answers

Answer:

The mass of the block is 1250g.

Explanation:

Given that the formula for density is ρ = mass/volume. Then you have to substitute the values into the formula :

[tex]ρ = \frac{mass}{volume} [/tex]

Let density = 250,

Let volume = 5,

[tex]250 = \frac{m}{5} [/tex]

[tex]m = 250 \times 5[/tex]

[tex]m = 1250g[/tex]

Assuming 100% efficient energy conversion, how much water stored behind a 50 centimeter high hydroelectric dam would be required to charge the battery?

Answers

Complete question is;

Assuming 100% efficient energy conversion how much water stored behind a 50 centimeter high hydroelectric dam would be required to charge the battery with power rating, 12 V, 50 Ampere-minutes.

Answer:

Amount of water required to charge the battery = 7.35 m³

Explanation:

The formula for Potential energy of the water at that height = mgh

Where;

m = mass of the water

g = acceleration due to gravity = 9.8 m/s²

h = height of water = 50 cm = 0.5 m

We know that in density, m = ρV

Where;

ρ = density of water = 1000 kg/m³

V = volume of water

So, potential energy is now given as;

Potential energy = ρVgH = 1000 × V × 9.8 × 0.5 = (4900V) J

Now, formula for energy of the battery is given as;

E = qV

We are given;

q = 50 A.min = 50 × 60 = 3,000 C

V = 12 V

Thus;

qV = 3,000 × 12 = 36,000 J

E = 36,000 J

At a 100% conversion rate, the energy of the water totally powers the battery.

Thus;

(4900V) = (36,000)

4900V = 36,000

V = 36,000/4900

V = 7.35 m³

EASY HELP
As a space shuttle climbs, _____.
its mass increases
its mass decreases
its weight increases
its weight decreases

Answers

Answer: it's weight decreases

Explanation:

A skateboarder, starting from rest, rolls down a 12.8-m ramp. When she arrives at the bottom of the ramp her speed is 8.89 m/s. (a) Determine the magnitude of her acceleration, assumed to be constant. (b) If the ramp is inclined at 32.6 ° with respect to the ground, what is the component of her acceleration that is parallel to the ground?

Answers

Answer:

a) a = 3.09 m/s²

b) aₓ = 2.60 m/s²

Explanation:

a) The magnitude of her acceleration can be calculated using the following equation:

[tex] V_{f}^{2} = V_{0}^{2} + 2ad [/tex]

Where:

[tex]V_{f}[/tex]: is the final speed = 8.89 m/s

[tex]V_{0}[/tex]: is the initial speed = 0 (since she starts from rest)

a: is the acceleration

d: is the distance = 12.8 m    

[tex] a = \frac{V_{f}^{2}}{2d} = \frac{(8.89 m/s)^{2}}{2*12.8 m} = 3.09 m/s^{2} [/tex]

Therefore, the magnitude of her acceleration is 3.09 m/s².              

b) The component of her acceleration that is parallel to the ground is given by:

[tex] a_{x} = a*cos(\theta) [/tex]

Where:

θ: is the angle respect to the ground = 32.6 °

[tex] a_{x} = 3.09 m/s^{2}*cos(32.6) = 2.60 m/s^{2} [/tex]

Hence, the component of her acceleration that is parallel to the ground is 2.60 m/s².

I hope it helps you!

A skateboarder, starting from rest, rolls down a 12.8-m ramp the magnitude of the skateboarder's acceleration is approximately 3.07 [tex]m/s^2[/tex], the component of her acceleration that is parallel to the ground is approximately 1.66 [tex]m/s^2[/tex].

(a) The following kinematic equation can be used to calculate the skateboarder's acceleration:

[tex]v^2 = u^2 + 2as[/tex]

[tex](8.89)^2 = (0)^2 + 2a(12.8)[/tex]

78.72 = 25.6a

a = 78.72 / 25.6

a = 3.07 [tex]m/s^2[/tex]

(b) Trigonometry can be used to calculate the part of her acceleration that is parallel to the ground. We are aware that the ramp's angle with the ground is 32.6°.

[tex]a_{parallel }= a * sin(\theta)[/tex]

Plugging in the values:

[tex]a_{parallel[/tex] = 3.07  [tex]m/s^2[/tex]* sin(32.6°)

[tex]a_{parallel[/tex]≈ 1.66  [tex]m/s^2[/tex]

Therefore, the component of her acceleration that is parallel to the ground is approximately 1.66  [tex]m/s^2[/tex].

For more details regarding acceleration, visit:

https://brainly.com/question/2303856

#SPJ6

John pushes Hector on a plastic toboggan.The free-body diagram is shown. A free body diagram with 4 force vectors. The first vector is pointing downward, labeled F Subscript g Baseline = negative 490 N. The second vector is pointing right, labeled F Subscript t Baseline = 735 N. The third vector is pointing upward, labeled F Subscript N Baseline = 490 N. The fourth vector is pointing left, labeled F Subscript f Baseline = negative 245 N. The up and down vectors are the same length. The right vector is longer than the left vector. What is the net force acting on Hector and the toboggan?

Answers

Answer:

490 N

Explanation:

is the correct answer

If the up and down vectors are the same length. The right vector is longer than the left vector, then  the net force acting on Hector and the toboggan would be 490 Newtons.

What is Newton's second law?

Newton's Second Law states that The resultant force acting on an object is proportional to the rate of change of momentum.

As given in the problem John pushes Hector on a plastic toboggan .The free-body diagram is shown. A free body diagram with 4 force vectors. The first vector is pointing downward, labeled F Subscript g Baseline = negative 490 N. The second vector is pointing right, labeled F Subscript t Baseline = 735 N. The third vector is pointing upward, labeled F Subscript N Baseline = 490 N. The fourth vector is pointing left, labeled F Subscript f Baseline = negative 245 N.

The net force acting on the vertical direction = 490-490

                                                                           =0

The net force acting on the horizontal direction = 735 -245

                                                                                =490 Newtons

Thus, the net force acting on Hector and the toboggan would be 490 Newtons.

Learn more about Newton's second law from here, refer to the link ;

brainly.com/question/13447525

#SPJ5

Other Questions
What is The mass of an electron A couple borrows $200,000 for a mortgage that requires fixed monthly payments over 30 consecutive years. The first monthly payment is due in one month. If the interest rate on the mortgage is 5%, which of the following comes closest to the monthly payment?When would the calculation of the effective annual interest rate be most useful?a. When comparing two investments with different annuity amountsb. When comparing two investments with different par valuesc. When comparing two investments that end at different points in timed. When comparing two investments that compound differently within a yeare. When comparing two investments that have different inherent risk A stone is thrown vertically into the air at an initial velocity of 79 ft/s. On a different planet, the height s (in feet) of the stone above the ground after t seconds is sequals79tminus3t squared and on Earth it is sequals79tminus16t squared. How much higher will the stone travel on the other planet than on Earth? Solve for y: 3y + 6 = -45 A researcher classifies firefighters according to whether their gloves fit well or poorly and by gender. They want to know if there is a difference in the proportion of poorly fitted gloves and gender. At alpha = 0.01, use the chi-square test to determine if there is a difference in the population proportion of glove fitness for the two genders. Observed data Males Females TotalGloves fit poorly 132 20 152Gloves fit well 415 19 434Total 547 39 586Expected data Males Females TotalGloves fit poorlyGloves fit wellTotal Write a short essay comparing and contrasting lyric and narrative poetry. A short-term effect of anorexia might include weight loss.O A. TrueO B. False Which of the steps of the scientific method means to create an explanation of the facts as you see them?a.Observec.Testb.Hypothesized.PredictPlease select the best answer from the choices providedHURRY, PLEASE HELP ME! a student placed a stack of rocks on a ramp. she added clumps of soil rooted with live grass. using a garden hose, she then applied a steady spray of water that soaked into the soil, making it heavier. what does the students experiment model Which inequality is true?5(2)>103(13) Government spending that is required by laws that have been previously passed is called __________ spending. What is the slope-intercept for Y-5 =-3x+2 How many square roots is the area ? Plaintiffs filed a class action lawsuit against investment banks alleging that they inflated prices on more than 300 IPOs, causing IPO investors to overpay for stock, and unlawfully benefited these banks through overcompensation of banking commissions and profits made through quick sales of this stock in their own accounts before prices settled into a more realistic valuation. How would you combat such abuses going forward Fill in the blanks with the correct word or phrase.The process of change in a population over time is called .The process by which organisms that are better adapted to the environment survive and reproduce is called . Please help me with 2 more questions ill give good ratings please! shoulders knees and toes wait i forgot the head,, thats what you'll get latersorry i just needed to put something here Select the correct answer. Neurofibromatosis, a genetic disease caused by mutation on the NF1 gene, follows an autosomal dominant pattern of inheritance. The diagram shows that the father carries two copies of the defective gene and the mother is unaffected. What percentage of offspring will have neurofibromatosis? a diagram showing a father who has the disease neurofibromatosis (alleles RR) and the mother is unaffected (alleles rr) A. 0% B. 25% C. 50% D. 100% CAN SOMEONE HELP ON MY LAST QUESTION PLZZZ How much work must be done on a 10 kg snowboard to increase its speed from 4 m/s to 6 m/s?